Difference between revisions of "2001 AIME I Problems/Problem 11"

(Solution)
Line 3: Line 3:
  
 
== Solution ==
 
== Solution ==
Let <math>P_{i} = (i,a_{i})</math>, where the first coordinate represents the row number and <math>a_i</math> represents the column number. Then <math>x_{1} = a_{1}</math>, <math>x_{2} = N + a_{2}</math>, <math>x_{3} = 2N + a_{3}</math>, etc. and <math>y_{1} = 5(a_{1} - 1) + 1</math>, <math>y_{2} = 5(a_{2} - 1) + 2</math>, etc. Now we get the system of equations:
 
<cmath>
 
\par \begin{align}a_{1} & = 5(a_{2} - 1) + 2 \\
 
N + a_{2} & = 5(a_{1} - 1) + 1 \\
 
2N + a_{3} & = 5(a_{4} - 1) + 4 \\
 
3N + a_{4} & = 5(a_{5} - 1) + 5 \\
 
4N + a_{5} & = 5(a_{3} - 1) + 3 \end{align}
 
</cmath>
 
We solve the system (the first two equations, and then the latter three) to get
 
<cmath>
 
\left(a_{1},a_{2},a_{3},a_{4},a_{5}\right) = \left(\frac {23 + 5N}{24},\frac {19 + N}{24},\frac {51 + 117N}{124},\frac {35 + 73N}{124},\frac {7 + 89N}{124}\right).
 
</cmath>
 
<math>a_{1},a_{2}</math> will be integers iff <math>N\equiv 5\pmod{24}</math> and <math>a_{3},a_{4},a_{5}</math> will be integers iff <math>N\equiv 25\pmod{124}</math>. Solving these [[Congruent (modular arithmetic)|congruence]]s simultaneously by standard methods gives <math>N\equiv \boxed{149}\pmod{744}</math>.
 
  
 
== See also ==
 
== See also ==

Revision as of 19:37, 26 April 2014

Problem

In a rectangular array of points, with 5 rows and $N$ columns, the points are numbered consecutively from left to right beginning with the top row. Thus the top row is numbered 1 through $N,$ the second row is numbered $N + 1$ through $2N,$ and so forth. Five points, $P_1, P_2, P_3, P_4,$ and $P_5,$ are selected so that each $P_i$ is in row $i.$ Let $x_i$ be the number associated with $P_i.$ Now renumber the array consecutively from top to bottom, beginning with the first column. Let $y_i$ be the number associated with $P_i$ after the renumbering. It is found that $x_1 = y_2,$ $x_2 = y_1,$ $x_3 = y_4,$ $x_4 = y_5,$ and $x_5 = y_3.$ Find the smallest possible value of $N.$

Solution

See also

2001 AIME I (ProblemsAnswer KeyResources)
Preceded by
Problem 10
Followed by
Problem 12
1 2 3 4 5 6 7 8 9 10 11 12 13 14 15
All AIME Problems and Solutions

The problems on this page are copyrighted by the Mathematical Association of America's American Mathematics Competitions. AMC logo.png